Page 1 of 1

PT 33 question section 1 #7

Posted: Fri May 15, 2015 5:21 pm
by ltowns1
Wait a minute....huh?? This is an argument???? I was looking on the Manhattan LR board, and LSAT hacks and they both say this is a argument?????? Is it because of the word "statements?? Not really following why this is an argument instead of an inference question

Re: PT 33 question section 1 #7

Posted: Fri May 15, 2015 6:49 pm
by giantswan
I'm a little confused by your phrasing (maybe just because I'm not familiar with Manhattan stuff). Are you asking why the stimulus is considered an argument?

If so, it has nothing to do with the word "statements" in the question stem. It is considered an argument because there is a conclusion ("That is why...").

Re: PT 33 question section 1 #7

Posted: Fri May 15, 2015 7:01 pm
by ltowns1
giantswan wrote:I'm a little confused by your phrasing (maybe just because I'm not familiar with Manhattan stuff). Are you asking why the stimulus is considered an argument?

If so, it has nothing to do with the word "statements" in the question stem. It is considered an argument because there is a conclusion ("That is why...").

Ok I see

Re: PT 33 question section 1 #7

Posted: Tue May 19, 2015 1:57 pm
by Christine (MLSAT)
I'd like to clarify this a little! The stimulus is an argument AND this is an inference question. Those two things are not mutually exclusive, though a lot of people think they are.

The question stem determines your task. For an 'assumption' question, you know right away that you are going to be dealing with an argument, and that you'll be finding the gap between the premise and the conclusion.

For an inference question, you're looking for something that must be true (or is extremely likely). Usually, you're inferring from a set of factual-sounding statements. Occasionally, though, you're given an argument. Now, on an inference question, you have to accept everything in the stimulus as true, right? So, if you accept an argument, that means you accept not only the premises, but also the conclusion and the reasoning used to get there as true.

And ultimately, that means that you have to accept any necessary assumptions of the argument also. If you didn't, then you couldn't possible accept the conclusion and reasoning, and you know you have to!

As a result, the correct answer on this question is both something that is very supported, if we accept the argument as valid and true, and also a 'necessary assumption' of the argument.

Re: PT 33 question section 1 #7

Posted: Tue May 19, 2015 4:28 pm
by ltowns1
Interesting,thanks as always!!!